Yahoo Answers is shutting down on May 4th, 2021 (Eastern Time) and beginning April 20th, 2021 (Eastern Time) the Yahoo Answers website will be in read-only mode. There will be no changes to other Yahoo properties or services, or your Yahoo account. You can find more information about the Yahoo Answers shutdown and how to download your data on this help page.

Find the number of shots required for the probability to hit the target becomes 0.9, if the chance of hitting the target is 0.4 per shot.?

What I tried to do is this:

I formed a series which describes the probability the gun will hit the target after each shot:

0.4 +0.6*0.4 +0.6*0.6*0.4 etc etc

But I do not know what method I can use to determine the LEAST number of terms which will add up to 0.9 or more.

3 Answers

Relevance
  • cidyah
    Lv 7
    6 years ago
    Favorite Answer

    sum of n terms of a geometric series 1+ r + r^2 + r^3 + ....+ r^n = (1-r^n) /(1-r)

    H = hits the target

    H' = does not hit the target

    P(H) + P(H')P(H) + P(H')P(H')P(H) + ..... = 0.9

    P(H) = 0.4

    .4 + (0.6)(0.4) + (0.6)^2 (0.4) +..... (0.6)^(n-1) (0.4) = 0.9

    0.4 ( 1+(0.6)+(0.6)^2 + .... +(0.6)^n ) = 0.9

    (0.4) ( 1- (0.6)^n) / (1-0.6) = 0.9

    1-(0.6)^n = 0.9

    -(0.6)^n = -0.1

    (0.6)^n = 0.1

    n log(0.6) = log(0.1)

    n = log(0.1) /log(0.6) = 4.5 = 5 (rounded)

  • Anonymous
    6 years ago

    chance of missing is .6

    chances of having zero hits after go n = .6^n

    so after 1 go you have a .6 chance of no hits, then .36 chance of no hits in 2 goes

    therefore after n goes, the chance of at least 1 hit is

    1 - (chance of no hits)

    =1 - .6^n

    need 1 - .6^n = .9

    .6^n = 1 - .9 = .1

    .6^n = .1

    n log(.6) = log(.1)

    n = log(.1) / log(.6) = 4.51

    4.5 shots isn't possible though, but if you remember thaAT we need

    1 - .6^n >= .9

    so after 4 goes

    1 - .6/^4 = .87

    after 5 goes

    1 - .6/^5 = ..92

    answer 5 goes

    1 - .9^4 =

  • 6 years ago

    You do want the probability of MISSING a lot of times to be reduced to 0.1, so you want

    (0.6)^N < 0.1. This isn't too hard:

    N*log(0.6) < -1

    N*(0.778 - 1) < -1

    Looks like N needs to be 5.

    Check: 0.6^4 = 0.1296 and 0.6^5 = about 0.08. Yes, 5 shots are needed.

Still have questions? Get your answers by asking now.